¿Por qué ciertas rotaciones son inestables? (Ecuaciones de Euler)

Tenemos las ecuaciones de Euler para un cuerpo giratorio de la siguiente manera

yo 1 ω ˙ 1 + ω 2 ω 3 ( yo 3 yo 2 ) = 0 yo 2 ω ˙ 2 + ω 1 ω 3 ( yo 1 yo 3 ) = 0 yo 3 ω ˙ 3 + ω 2 ω 1 ( yo 2 yo 1 ) = 0
Donde yo i son los momentos de inercia con respecto a la X i eje, y ω i es la velocidad angular alrededor de este eje.

Se puede demostrar (*) que si yo 1 > yo 2 > yo 3 , entonces objetos con velocidad angular muy cerca de ω = ( 0 , 1 , 0 ) son inestables ¿Por qué es esto y cómo puedo tratar de imaginarlo?

Traté de imaginar esto usando una pelota, pero me di cuenta de que probablemente no sea una buena forma de visualizarlo, ya que una pelota es esféricamente simétrica, por lo que los momentos de inercia no son distintos. ¿Hay alguna visualización o animación que me permita ver esta rotación y posiblemente entender por qué es inestable?


(*) En respuesta al comentario de @SRS:

No estoy seguro de ninguna referencia, pero sé cómo hacerlo: Let ω 1 = η 1 , ω 3 = η 3 donde η es una pequeña perturbación, y supongamos ω 2 = 1 + η 2 . Entonces los ecuaciones de Euler se convierten en

(1) yo 1 η ˙ 1 = ( yo 2 yo 3 ) η 3 + O ( η 2 )
(2) yo 2 η ˙ 2 = O ( η 2 )
(3) yo 3 η ˙ 3 = ( yo 1 yo 2 ) η 1 + O ( η 2 )
Diferenciar ( 1 ) y sub en ( 3 ) a la expresión resultante
η ¨ 1 = ( yo 2 yo 3 ) ( yo 1 yo 2 ) yo 3 yo 1 η 1
Si yo 1 > yo 2 > yo 3 , entonces la constante del lado derecho es positiva, por lo que la solución a esta ecuación es exponencial (si fuera de cualquier otro orden, entonces la solución sería una pecado / porque ). Por lo tanto, es inestable.


Editar:

Para aclarar, publiqué esta pregunta para ver otras formas más visuales de comprender este efecto en lugar de resolver las ecuaciones como lo hice anteriormente, y para ver cómo este efecto entra en juego en la vida real. Entonces, no creo que sea un duplicado de las otras preguntas, ya que no tienen respuestas que se ajusten a esto.

Lo acabas de demostrar jaja. SI quieres probarlo, la mejor manera es, como dices, elegir un objeto con yo 1 yo 2 yo 3 yo 1 , y un buen 1 es una caja de pañuelos, o cualquier prisma.
@FGSUZ Sí, sé cómo mostrarlo matemáticamente, mi pregunta es sobre cómo visualizarlo, como los enlaces proporcionados por ZeroTheHero
También puedes demostrarlo tú mismo con un zapato.
@Qmechanic: estoy de acuerdo en que este es un duplicado de esas preguntas anteriores, pero me gustan las respuestas aquí más que a cualquiera de esas otras. Quizás esas preguntas más antiguas deberían cerrarse como un duplicado de esto.
@Qmechanic Ya había visto una de esas preguntas cuando busqué, pero no había encontrado las otras. De todos modos, no creo que sea un duplicado porque pedí específicamente una forma más visual de entender esto, en lugar de simplemente mostrar que una pequeña perturbación conduce a una solución exponencial.
Otra buena visualización es considerar la rotación de una barra larga alrededor de un eje cercano, pero no igual a su eje central. Cuando esté girando exactamente sobre ese eje central, será constante. Pero si está un poco desviado, las fuerzas de rotación alejarán la varilla de ese eje.

Respuestas (3)

Hay otra buena manera de ver esto matemáticamente. No es demasiado difícil demostrar que en la estructura del cuerpo hay dos cantidades conservadas: el cuadrado del vector de momento angular

L 2 = L 1 2 + L 2 2 + L 3 2
y la energía cinética de rotación, que resulta ser
T = 1 2 ( L 1 2 yo 1 + L 2 2 yo 2 + L 3 2 yo 3 ) .
(Tenga en cuenta que el momento angular L en sí mismo no se conserva en la estructura del cuerpo; pero resulta que su cuadrado es una constante.)

Entonces podemos hacer la pregunta: Para valores dados de L 2 y T , ¿cuáles son los valores permitidos de L ? Es fácil ver eso L 2 restricción significa que L debe estar en la superficie de una esfera; y es casi tan fácil ver que el T restricción significa que L también debe estar en la superficie de un elipsoide dado, con ejes principales 2 T yo 1 > 2 T yo 2 > 2 T yo 3 . Por lo tanto, los valores permitidos de L debe estar en la intersección de una esfera y un elipsoide. si sostenemos L 2 fijo y generar un montón de estas curvas para varios valores de T , se ven así:

ingrese la descripción de la imagen aquí

Tenga en cuenta que para un valor dado de L 2 , un objeto tendrá su energía cinética más alta posible cuando gira alrededor del eje con el momento de inercia más bajo , y viceversa.

Supongamos, entonces, que un objeto gira alrededor del eje de su momento de inercia más alto. Si perturbamos este objeto de modo que cambiemos ligeramente su energía (suponiendo por el bien del argumento que L 2 permanece constante), vemos que el vector L ahora estará en una curva relativamente pequeña cerca de su ubicación original. De manera similar, si el objeto gira alrededor de su eje de menor inercia, L permanecerá relativamente cerca de su valor original cuando sea perturbado.

Sin embargo, la situación es marcadamente diferente cuando el objeto está girando inicialmente sobre el eje intermedio (el tercer punto rojo en el diagrama de arriba, en el "lado frontal" de la esfera. Los contornos de ligeramente perturbados T cerca de este punto no te quedes cerca del eje intermedio; vagan por toda la esfera. Por lo tanto, nada guarda L de deambular por toda esta esfera si perturbamos el objeto ligeramente para que no gire alrededor de este eje; lo que implica que un objeto que gira sobre su eje intermedio es inestable.

No lo había visto así antes. Todavía encuentro el diagrama bastante abstracto, pero después de ver los videos sobre las cosas de la barra en T, se vuelve un poco más claro: las curvas siguen las líneas más grandes en forma de elipse para las oscilaciones inestables. ¡Gracias! :)
Hermosa respuesta.
Estas respuestas, la tuya y la de ZeroTheHero, son simplemente hermosas. Nunca antes lo había pensado en estos términos: simplemente me alegró saber que una linealización estable de un sistema alrededor de un punto de equilibrio significa que uno puede encontrar una bola de radio distinto de cero en la que uno puede reformular el problema como un fijo contractivo. problema de puntos. Pero, ¡tanta belleza desde la perspectiva global también!
Hola Michael, estaba jugando tratando de escribir un programa para producir una trama como esta. Logré obtener uno que se parece a la trama de ZeroTheHero, pero no puedo encontrar la forma de trazar tu figura. ¿Calculó la forma general de la ecuación de las curvas amarillas y luego las graficó? Si es así, ¿recuerdas cuáles eran?
@JohnDoe: Para esa versión de la trama, en realidad trabajé explícitamente en la forma de las curvas; sin embargo, en realidad no caben en este cuadro de comentarios. :-) Efectivamente, sin embargo, expresas L 1 , L 2 , L 3 en coordenadas esféricas, escriba ambas superficies en términos de coordenadas esféricas, iguale entre sí, use la identidad pecado 2 θ = ( 1 porque 2 θ ) , y luego resuelve para porque 2 θ como una función de ϕ . Sin embargo, si está usando Mathematica, una forma más fácil de hacerlo es usando la MeshFunctionsopción, con la energía cinética como su MeshFunction.

Hay una alternativa al método de @MichaelSeifert que utiliza el momento angular y los momentos de inercia: es tratar con el vector ω directamente ya que estamos interesados ​​en la evolución de este vector.

Se puede expresar la energía cinética y la longitud al cuadrado de L como

T = 1 2 ( yo 1 ω 1 2 + yo 2 ω 2 2 + yo 3 ω 3 2 ) L L = yo 1 2 ω 1 2 + yo 2 2 ω 2 2 + yo 3 2 ω 3 2
Estas dos ecuaciones definen elipsoides distintos en ω -espacio. Como hay tres componentes para ω pero solo dos restricciones, las restricciones no son suficientes para determinar completamente ω . Más bien, la evolución de ω debe estar en la línea de intersección de estas dos elipses, como lo ilustra la línea negra en la figura. (La elipse verde es constante L L y el amarillo constante T .)

ingrese la descripción de la imagen aquí

Esta intersección aquí tiene aproximadamente la forma de un plátano aplanado, con el lado largo del plátano en el ω 2 dirección.

Así, durante la evolución, la punta de ω puede moverse a lo largo de esta intersección mientras mantiene la energía y L L constante. La figura ilustra el caso donde

yo 1 = 4 , yo 2 = 3 / 2 , yo 3 = 1
con T = 5 / 2 y | L | = 2.45 .

Puede ver en la figura que, si comenzamos en la intersección en la parte superior, la evolución de ω 3 tiene un rango bastante restringido (y no puede cambiar de signo), que la evolución de ω 1 también es bastante restringido, pero que el componente ω 2 es mucho más grande (básicamente toda la longitud del plátano aplastado). Por lo tanto, cualitativamente hablando , la rotación alrededor del eje medio es inestable porque ω 2 no necesita permanecer cerca de su valor inicial.

(El mismo argumento funciona para los componentes L k = ω k yo k en la figura de MichaelSeifert excepto que, en su caso, L 1 no cambiaria de signo pero L 3 podría cambiar de signo cerca del punto fijo hiperbólico.)

Y esta es una hermosa respuesta también. Por cierto, una vez que aprendas esto, y lo pruebes en un libro alrededor de su eje medio, y lo veas dar vueltas, nunca lo olvidarás. El mejor libro para dar vueltas es Mecánica clásica de Goldstein.
Estas respuestas, la tuya y la de Michael, son simplemente hermosas. Nunca antes lo había pensado en estos términos: simplemente me alegró saber que una linealización estable de un sistema alrededor de un punto de equilibrio significa que uno puede encontrar una bola de radio distinto de cero en la que uno puede reformular el problema como un fijo contractivo. problema de puntos. Pero, ¡tanta belleza desde la perspectiva global también!
De hecho, esta es la forma más tradicional de hacerlo y, en este caso, las curvas de intersección se denominan polhodes (una de mis palabras físicas favoritas). Me resulta un poco más fácil de visualizar cuando una de las superficies restringidas es una esfera. .
@MichaelSeifert Aprendí una cosa más hoy. Gracias.

Mi profesor de física nos ayudó a visualizar esto con una raqueta de tenis.

Girar con el eje a lo largo del mango es estable.

El giro en el plano de la raqueta es estable. John McEnroe volteaba mucho sus raquetas así.

Girar en la otra dirección, como cuando se mueve una raqueta, es inestable. No importa cuán cuidadosamente voltees la raqueta, también gira en la otra dirección antes de volver a tu mano.

En caso de no tener acceso a una raqueta para jugar: https://www.youtube.com/watch?v=4dqCQqI-Gis

¡Gracias! Hasta que hice esta pregunta, no me di cuenta de que esto a veces se llama el teorema de la raqueta de tenis. Saber eso hubiera hecho que visualizarlo en la vida real fuera mucho más fácil.